Are you seeking one-on-one college counseling and/or essay support? Limited spots are now available. Click here to learn more.

How to Write the AP Lang Rhetorical Analysis Essay (With Example)

November 27, 2023

Feeling intimidated by the AP Lang Rhetorical Analysis Essay? We’re here to help demystify. Whether you’re cramming for the AP Lang exam right now or planning to take the test down the road, we’ve got crucial rubric information, helpful tips, and an essay example to prepare you for the big day. This post will cover 1) What is the AP Lang Rhetorical Analysis Essay? 2) AP Lang Rhetorical Analysis Rubric 3) AP Lang Rhetorical Analysis: Sample Prompt 4) AP Lang Rhetorical Analysis Essay Example 5)AP Lang Rhetorical Analysis Essay Example: Why It Works

What is the AP Lang Rhetorical Analysis Essay?

The AP Lang Rhetorical Analysis Essay is one of three essays included in the written portion of the AP English Exam. The full AP English Exam is 3 hours and 15 minutes long, with the first 60 minutes dedicated to multiple-choice questions. Once you complete the multiple-choice section, you move on to three equally weighted essays that ask you to synthesize, analyze, and interpret texts and develop well-reasoned arguments. The three essays include:

Synthesis essay: You’ll review various pieces of evidence and then write an essay that synthesizes (aka combines and interprets) the evidence and presents a clear argument. Read our write up on How to Write the AP Lang Synthesis Essay here.

Argumentative essay: You’ll take a stance on a specific topic and argue your case.

Rhetorical essay: You’ll read a provided passage, then analyze the author’s rhetorical choices and develop an argument that explains why the author made those rhetorical choices.

AP Lang Rhetorical Analysis Rubric

The AP Lang Rhetorical Analysis Essay is graded on just 3 rubric categories: Thesis, Evidence and Commentary, and Sophistication . At a glance, the rubric categories may seem vague, but AP exam graders are actually looking for very particular things in each category. We’ll break it down with dos and don’ts for each rubric category:

Thesis (0-1 point)

There’s nothing nebulous when it comes to grading AP Lang Rhetorical Analysis Essay thesis. You either have one or you don’t. Including a thesis gets you one point closer to a high score and leaving it out means you miss out on one crucial point. So, what makes a thesis that counts?

  • Make sure your thesis argues something about the author’s rhetorical choices. Making an argument means taking a risk and offering your own interpretation of the provided text. This is an argument that someone else might disagree with.
  • A good test to see if you have a thesis that makes an argument. In your head, add the phrase “I think that…” to the beginning of your thesis. If what follows doesn’t logically flow after that phrase (aka if what follows isn’t something you and only you think), it’s likely you’re not making an argument.
  • Avoid a thesis that merely restates the prompt.
  • Avoid a thesis that summarizes the text but does not make an argument.

Evidence and Commentary (0-4 points)

This rubric category is graded on a scale of 0-4 where 4 is the highest grade. Per the AP Lang Rhetorical Analysis rubric, to get a 4, you’ll want to:

  • Include lots of specific evidence from the text. There is no set golden number of quotes to include, but you’ll want to make sure you’re incorporating more than a couple pieces of evidence that support your argument about the author’s rhetorical choices.
  • Make sure you include more than one type of evidence, too. Let’s say you’re working on your essay and have gathered examples of alliteration to include as supporting evidence. That’s just one type of rhetorical choice, and it’s hard to make a credible argument if you’re only looking at one type of evidence. To fix that issue, reread the text again looking for patterns in word choice and syntax, meaningful figurative language and imagery, literary devices, and other rhetorical choices, looking for additional types of evidence to support your argument.
  • After you include evidence, offer your own interpretation and explain how this evidence proves the point you make in your thesis.
  • Don’t summarize or speak generally about the author and the text. Everything you write must be backed up with evidence.
  • Don’t let quotes speak for themselves. After every piece of evidence you include, make sure to explain your interpretation. Also, connect the evidence to your overarching argument.

Sophistication (0-1 point)

In this case, sophistication isn’t about how many fancy vocabulary words or how many semicolons you use. According to College Board , one point can be awarded to AP Lang Rhetorical Analysis essays that “demonstrate sophistication of thought and/or a complex understanding of the rhetorical situation” in any of these three ways:

  • Explaining the significance or relevance of the writer’s rhetorical choices.
  • Explaining the purpose or function of the passage’s complexities or tensions.
  • Employing a style that is consistently vivid and persuasive.

Note that you don’t have to achieve all three to earn your sophistication point. A good way to think of this rubric category is to consider it a bonus point that you can earn for going above and beyond in depth of analysis or by writing an especially persuasive, clear, and well-structured essay. In order to earn this point, you’ll need to first do a good job with your thesis, evidence, and commentary.

  • Focus on nailing an argumentative thesis and multiple types of evidence. Getting these fundamentals of your essay right will set you up for achieving depth of analysis.
  • Explain how each piece of evidence connects to your thesis.
  • Spend a minute outlining your essay before you begin to ensure your essay flows in a clear and cohesive way.
  • Steer clear of generalizations about the author or text.
  • Don’t include arguments you can’t prove with evidence from the text.
  • Avoid complex sentences and fancy vocabulary words unless you use them often. Long, clunky sentences with imprecisely used words are hard to follow.

AP Lang Rhetorical Analysis: Sample Prompt

The sample prompt below is published online by College Board and is a real example from the 2021 AP Exam. The prompt provides background context, essay instructions, and the text you need to analyze. For sake of space, we’ve included the text as an image you can click to read. After the prompt, we provide a sample high scoring essay and then explain why this AP Lang Rhetorical Analysis essay example works.

Suggested time—40 minutes.

(This question counts as one-third of the total essay section score.)

On February 27, 2013, while in office, former president Barack Obama delivered the following address dedicating the Rosa Parks statue in the National Statuary Hall of the United States Capitol building. Rosa Parks was an African American civil rights activist who was arrested in 1955 for refusing to give up her seat on a segregated bus in Montgomery, Alabama. Read the passage carefully. Write an essay that analyzes the rhetorical choices Obama makes to convey his message.

In your response you should do the following:

  • Respond to the prompt with a thesis that analyzes the writer’s rhetorical choices.
  • Select and use evidence to support your line of reasoning.
  • Explain how the evidence supports your line of reasoning.
  • Demonstrate an understanding of the rhetorical situation.
  • Use appropriate grammar and punctuation in communicating your argument.

AP Lang Rhetorical Analysis Essay Example

In his speech delivered in 2013 at the dedication of Rosa Park’s statue, President Barack Obama acknowledges everything that Parks’ activism made possible in the United States. Telling the story of Parks’ life and achievements, Obama highlights the fact that Parks was a regular person whose actions accomplished enormous change during the civil rights era. Through the use of diction that portrays Parks as quiet and demure, long lists that emphasize the extent of her impacts, and Biblical references, Obama suggests that all of us are capable of achieving greater good, just as Parks did.

Although it might be a surprising way to start to his dedication, Obama begins his speech by telling us who Parks was not: “Rosa Parks held no elected office. She possessed no fortune” he explains in lines 1-2. Later, when he tells the story of the bus driver who threatened to have Parks arrested when she refused to get off the bus, he explains that Parks “simply replied, ‘You may do that’” (lines 22-23). Right away, he establishes that Parks was a regular person who did not hold a seat of power. Her protest on the bus was not part of a larger plan, it was a simple response. By emphasizing that Parks was not powerful, wealthy, or loud spoken, he implies that Parks’ style of activism is an everyday practice that all of us can aspire to.

AP Lang Rhetorical Analysis Essay Example (Continued)

Even though Obama portrays Parks as a demure person whose protest came “simply” and naturally, he shows the importance of her activism through long lists of ripple effects. When Parks challenged her arrest, Obama explains, Martin Luther King, Jr. stood with her and “so did thousands of Montgomery, Alabama commuters” (lines 27-28). They began a boycott that included “teachers and laborers, clergy and domestics, through rain and cold and sweltering heat, day after day, week after week, month after month, walking miles if they had to…” (lines 28-31). In this section of the speech, Obama’s sentences grow longer and he uses lists to show that Parks’ small action impacted and inspired many others to fight for change. Further, listing out how many days, weeks, and months the boycott lasted shows how Parks’ single act of protest sparked a much longer push for change.

To further illustrate Parks’ impact, Obama incorporates Biblical references that emphasize the importance of “that single moment on the bus” (lines 57-58). In lines 33-35, Obama explains that Parks and the other protestors are “driven by a solemn determination to affirm their God-given dignity” and he also compares their victory to the fall the “ancient walls of Jericho” (line 43). By of including these Biblical references, Obama suggests that Parks’ action on the bus did more than correct personal or political wrongs; it also corrected moral and spiritual wrongs. Although Parks had no political power or fortune, she was able to restore a moral balance in our world.

Toward the end of the speech, Obama states that change happens “not mainly through the exploits of the famous and the powerful, but through the countless acts of often anonymous courage and kindness” (lines 78-81). Through carefully chosen diction that portrays her as a quiet, regular person and through lists and Biblical references that highlight the huge impacts of her action, Obama illustrates exactly this point. He wants us to see that, just like Parks, the small and meek can change the world for the better.

AP Lang Rhetorical Analysis Essay Example: Why It Works

We would give the AP Lang Rhetorical Analysis essay above a score of 6 out of 6 because it fully satisfies the essay’s 3 rubric categories: Thesis, Evidence and Commentary, and Sophistication . Let’s break down what this student did:

The thesis of this essay appears in the last line of the first paragraph:

“ Through the use of diction that portrays Parks as quiet and demure, long lists that emphasize the extent of her impacts, and Biblical references, Obama suggests that all of us are capable of achieving greater good, just as Parks did .”

This student’s thesis works because they make a clear argument about Obama’s rhetorical choices. They 1) list the rhetorical choices that will be analyzed in the rest of the essay (the italicized text above) and 2) include an argument someone else might disagree with (the bolded text above).

Evidence and Commentary:

This student includes substantial evidence and commentary. Things they do right, per the AP Lang Rhetorical Analysis rubric:

  • They include lots of specific evidence from the text in the form of quotes.
  • They incorporate 3 different types of evidence (diction, long lists, Biblical references).
  • After including evidence, they offer an interpretation of what the evidence means and explain how the evidence contributes to their overarching argument (aka their thesis).

Sophistication

This essay achieves sophistication according to the AP Lang Rhetorical Analysis essay rubric in a few key ways:

  • This student provides an introduction that flows naturally into the topic their essay will discuss. Before they get to their thesis, they tell us that Obama portrays Parks as a “regular person” setting up their main argument: Obama wants all regular people to aspire to do good in the world just as Rosa Parks did.
  • They organize evidence and commentary in a clear and cohesive way. Each body paragraph focuses on just one type of evidence.
  • They explain how their evidence is significant. In the final sentence of each body paragraph, they draw a connection back to the overarching argument presented in the thesis.
  • All their evidence supports the argument presented in their thesis. There is no extraneous evidence or misleading detail.
  • They consider nuances in the text. Rather than taking the text at face value, they consider what Obama’s rhetorical choices imply and offer their own unique interpretation of those implications.
  • In their final paragraph, they come full circle, reiterate their thesis, and explain what Obama’s rhetorical choices communicate to readers.
  • Their sentences are clear and easy to read. There are no grammar errors or misused words.

AP Lang Rhetorical Analysis Essay—More Resources

Looking for more tips to help your master your AP Lang Rhetorical Analysis Essay? Brush up on 20 Rhetorical Devices High School Students Should Know and read our Tips for Improving Reading Comprehension . If you’re ready to start studying for another part of the AP English Exam, find more expert tips in our How to Write the AP Lang Synthesis blog post.

Considering what other AP classes to take? Read up on the Hardest AP Classes .

  • High School Success

Christina Wood

Christina Wood holds a BA in Literature & Writing from UC San Diego, an MFA in Creative Writing from Washington University in St. Louis, and is currently a Doctoral Candidate in English at the University of Georgia, where she teaches creative writing and first-year composition courses. Christina has published fiction and nonfiction in numerous publications, including The Paris Review , McSweeney’s , Granta , Virginia Quarterly Review , The Sewanee Review , Mississippi Review , and Puerto del Sol , among others. Her story “The Astronaut” won the 2018 Shirley Jackson Award for short fiction and received a “Distinguished Stories” mention in the 2019 Best American Short Stories anthology.

  • 2-Year Colleges
  • Application Strategies
  • Best Colleges by Major
  • Best Colleges by State
  • Big Picture
  • Career & Personality Assessment
  • College Essay
  • College Search/Knowledge
  • College Success
  • Costs & Financial Aid
  • Data Visualizations
  • Dental School Admissions
  • Extracurricular Activities
  • Graduate School Admissions
  • High Schools
  • Homeschool Resources
  • Law School Admissions
  • Medical School Admissions
  • Navigating the Admissions Process
  • Online Learning
  • Outdoor Adventure
  • Private High School Spotlight
  • Research Programs
  • Summer Program Spotlight
  • Summer Programs
  • Teacher Tools
  • Test Prep Provider Spotlight

“Innovative and invaluable…use this book as your college lifeline.”

— Lynn O'Shaughnessy

Nationally Recognized College Expert

College Planning in Your Inbox

Join our information-packed monthly newsletter.

What are your chances of acceptance?

Calculate for all schools, your chance of acceptance.

Duke University

Your chancing factors

Extracurriculars.

rhetorical analysis ap lang example essay

How to Write the AP Lang Rhetorical Essay

Do you know how to improve your profile for college applications.

See how your profile ranks among thousands of other students using CollegeVine. Calculate your chances at your dream schools and learn what areas you need to improve right now — it only takes 3 minutes and it's 100% free.

Show me what areas I need to improve

What’s Covered:

What is the ap lang rhetorical essay, tips for writing the ap lang rhetorical essay.

  • AP Lang Rhetorical Essay Example

How Will AP Scores Affect College Chances?

The AP English Language Exam is one of the most common AP exams you can take. However, the average score on the exam in 2020 was a 2.96 out of 5. While this may seem a bit low, it is important to note that over 550,000 students take the exam annually. With some preparation and knowing how to study, it is totally possible to do well on this AP exam.

The AP Lang Rhetorical Essay is one section of the AP English Language Exam. The exam itself is 3 hours and 15 minutes long, and is broken into two sections. The first part of the exam is a 60 minute, 45-question multiple-choice section. The questions on this part of the exam will test your ability to read a passage and then interpret its meaning, style, and overall themes. After the multiple-choice section, there is a section lasting 2 hours and 15 minutes with three “free response” essays. This includes the synthesis essay, the rhetorical analysis essay, and the argument essay. 

  • In the synthesis essay , you will have to develop an argument using pieces of evidence provided to you. 
  • The argumentative essay will have you pick a side in a debate and argue for or against it.
  • The rhetorical essay requires that you discuss how an author’s written passage contributes to a greater meaning or theme. 

The rhetorical essay is perhaps the most unique of all AP Lang exam essays because it requires the test taker to analyze and interpret the deeper meanings of the passage and connect them to the author’s writing style and writing syntax in only 40 minutes. This essay can be the trickiest because it requires you to have knowledge of rhetorical strategies and then apply them to a passage you’ve never seen before.

1. Outline Your Essay Before Writing

One of the most important parts of the AP Lang essays is structuring your essay so that it makes sense to the reader. This is just as important as having good content. For this essay in particular, you’ll want to read the passage first and write a brief outline of your points before you begin the essay. This is because you will want to write the essay using the passage chronologically, which will be discussed in detail below.

2. Understand Rhetorical Strategies 

If you feel like you don’t know where to start as you prepare to study for the rhetorical essay portion of the exam, you aren’t alone. It is imperative that you have a grasp on what rhetorical strategies are and how you can use them in your essay. One definition of rhetoric is “language carefully chosen and arranged for maximum effect.” This can include types of figurative language (metaphor, simile, personification, pun, irony, etc.) elements of syntax (parallelism, juxtaposition, anthesis, anaphora, etc), logical fallacies, or persuasive appeals. Overall, there are many elements that you can analyze in an essay and having a good grasp on them through practice and memorization is important.

3. Keep the Essay Well Structured 

Even if you understand the various rhetorical strategies you can use, where do you begin? First of all, you’ll want to write a strong introduction that outlines the purpose of the piece. At the end of this introduction, you will write a thesis statement that encapsulates all the rhetorical strategies you discuss. Perhaps these are style elements, tone, or syntax. Be sure to be specific as you list these.

Next, you will create your body paragraphs. As you discuss the rhetorical elements in the piece and tie them back to the work’s meanings, be sure to discuss the points in chronological order. You don’t have to discuss every single strategy, but just pick the ones that are most important. Be sure to cite the line where you found the example. At the end of the essay, write a short conclusion that summarizes the major points above.

4. Be Sure to Explain Your Examples

As you write the essay, don’t just list out your examples and say something like “this is an example of ethos, logos, pathos.” Instead, analyze how the example shows that rhetoric device and how it helps the author further their argument. As you write the rhetorical essay, you’ll want to be as specific and detail-focused as possible. 

rhetorical analysis ap lang example essay

Discover your chances at hundreds of schools

Our free chancing engine takes into account your history, background, test scores, and extracurricular activities to show you your real chances of admission—and how to improve them.

AP Lang Rhetorical Analysis Essay Example

Below is a prompt and example for a rhetorical essay, along with its score and what the writer did well and could have improved:

The passage below is an excerpt from “On the Want of Money,” an essay written by nineteenth-century author William Hazlitt. Read the passage carefully. Then write an essay in which you analyze the rhetorical strategies Hazlitt uses to develop his position about money.

rhetorical analysis ap lang example essay

Student essay example:

In his essay, Hazlitt develops his position on money through careful use of adjectives and verbs, hypothetical situations, and images. His examples serve to impress upon the reader the highly negative consequences of being in “want of money.”

Hazlitt’s word choice in his opening phrase provides an example of his technique in the rest of the essay. It is not necessary to follow “literally” with “truly” yet his repetition of the same ideas emphasizes his point. In his next sentence, one that lasts forty-six lines, Hazlitt condignly repeats similar ideas, beating into his audience the necessity of having money in this world. The parallelism throughout that one long sentence, “it is not to be sent for to court, or asked out to dinner…it is not to have your own opinion consulted or sees rejected with contempt..” ties the many different situations Haziltt gives together. What could have become a tedious spiel instead becomes a melodious recitation, each example reminding you of one before it, either because of the similarities in structure or content. Hazlitt addresses many different negative effects of not having money but manages to tie them together with his rhetorical strategies. 

The diction of the passage fully relays Hazlitt’s position about money. In every example he gives a negative situation but in most emphasizes the terrible circumstance with strong negative adjectives or verbs. “Rejected,” “contempt,” “disparaged,” “scrutinized,” “irksome,” “deprived,” “assailed” “chagrin;” the endless repetition of such discouragement shows how empathetically Hazlitt believes money is a requisite for a happy life. Even the irony of the last sentences is negative, conveying the utter hopelessness of one without money. Through one may have none in life, pitiless men will proceed to mock one’s circumstances, “at a considerable expense” after death! 

In having as the body of his essay one long sentence, Hazlitt creates a flow that speeds the passage along, hardly giving the reader time to absorb one idea before another is thrown at him. The unceasing flow is synonymous with Hazlitt’s view of the life of a person without money: he will be “jostled” through life, unable to stop and appreciate the beauty around him or to take time for his own leisure. 

The score on this essay was a 6 out of 6. This essay started out very strong as the student had a concrete thesis statement explaining the strategies that Hazlitt used to develop his position on money as well as Hazlitt’s belief on the topic. In the thesis statement, the student points out that adjectives, verbs, hypothetical situations, and images help prove Hazlitt’s point that wanting money can be problematic. 

Next, the student broke down their points into three main subsections related to their thesis. More specifically, the student first discusses word choice of repetition and parallelism. When the student discusses these strategies, they list evidence in the paragraph that can be found chronologically in Hazlitt’s essay. The next paragraph is about diction, and the student used specific adjectives and verbs that support this idea. In the last paragraph, the student emphasized how the speed and flow of the essay helped describe Hazlitt’s viewpoint on life. This last concluding sentence is particularly thoughtful, as it goes beyond the explicit points made in the essay and discusses the style and tone of the writing. 

It is important to remember that in some ways, the rhetorical essay is also an argumentative essay, as the student must prove how certain rhetorical strategies are used and their significance in the essay. The student even discussed the irony of the paragraph, which is not explicit in the passage.

Overall, this student did an excellent job organizing and structuring the essay and did a nice job using evidence to prove their points. 

Now that you’ve learned about the AP Lang rhetorical essay, you may be wondering how your AP scores impact your chances of admission. In fact, your AP scores have relatively little impact on your admissions decision , and your course rigor has much more weight in the application process.

If you’d like to know your chances of admission, be sure to check out our chancing calculator! This tool takes into account your classes, extracurriculars, demographic information, and test scores to understand your chances at admission at over 600 schools. Best of all, it is completely free!

rhetorical analysis ap lang example essay

Related CollegeVine Blog Posts

rhetorical analysis ap lang example essay

pep

Find what you need to study

Rhetorical Analysis Essay How-To

7 min read • june 18, 2024

Brandon Wu

Kathryn Howard

What is the Rhetorical Analysis Essay?

The is a free-response question in which you analyze any rhetorical strategies that the author uses in the given passage and evaluate/discuss them in a well-formatted essay. 

According to the College Board:

 “The rhetorical analysis free-response essay question presents students with a passage of nonfiction prose of approximately 600 to 800 words. Students are asked to write an essay that analyzes the writer’s rhetorical choices . This question assesses students’ ability to do the following:

  • Respond to the prompt with a thesis that analyzes the writer’s rhetorical choices .
  • Select and use evidence to support your line of reasoning .
  • Explain how the evidence supports your line of reasoning .
  • Demonstrate an understanding of the rhetorical situation .
  • Use appropriate grammar and punctuation in communicating your argument." Essentially, you are being asked to analyze someone’s writing and what strategies they used to help them achieve their purpose.

Rhetorical Analysis Rubric for Scoring

The rhetorical analysis frq is out of 6 points.

✍ The Thesis of your for AP Lang can score you 1 point right from the start!
* Lacks a The thesis provides summary with no claim*They have a thesis, but it is not in response to the given prompt Student has a justifiable thesis that answers the prompt that analyzes the rhetorical strategies.

rhetorical analysis ap lang example essay

To get this point you need to clearly write a defensible thesis about  the rhetorical choices the author makes. Do not take a stance on the argument the author is making if he/she is making one. You are only talking about rhetorical strategies. 

🎥 Watch: AP Language -   How to Find Rhetorical Strategies

Evidence and Commentary (4 points) 📂

 Only restates thesis, has a lot of repetition, and the information doesn't have to do with the prompt : Provides that is mostly general.  : Summarizes but doesn't explain how it applies : Provides some pertinent .  Explains how some of the provided supports the student’s thesis, but does not have good reasoning : Provides specific to support all assertions made Explains how some of the supports a . AND Explains how at least one rhetorical choice in the excerpt helps to achieve the author’s purpose   : Provides specific to support all claims in a .  Consistently explains how the supports a . AND Explains how multiple rhetorical strategies help to achieve the author’s purpose.

To get the four points you need to  not only present evidence but  explain why  it supports your thesis and how it contributes to the author’s message.

Sophistication (1 point) 👓

 Does not meet all the standards for one point. Shows sophistication of thought and/or a of the . 

To get to this point you have to demonstrate a  complex understanding of  both what that purpose was, and how the rhetorical analysis devices aided the author’s purpose.

There are a few ways that you can earn the sophistication point :

  • Explaining the significance or relevance of the writer’s rhetorical choices (given the rhetorical situation ).
  • Explaining a purpose or function of the passage’s complexities or tensions.
  • Employing a style that is consistently vivid and persuasive.

You have 40 minutes to complete the rhetorical analysis essay for AP Lang:

  • 12 minutes: Read the text and plan out your essay. (TOBI)
  • 6 minutes: Write your introduction paragraph.
  • 18 minutes: Write 2-3 body paragraphs.
  • 2 minutes: Write a quick conclusion.
  • 2 minutes: Proofread and revise your essay. 🎥 Watch: AP Language -   Rhetorical Analysis Organization and Timing

rhetorical analysis ap lang example essay

How to Maximize Your Time⌚

Outline your rhetorical analysis essay before writing! A great tool for this is a TOBI:

TOBI stands for  thesis *,*  outline *,* and  big idea.

TOBI Outline

  • BI -Big Idea Here is an example of how to use TOBI given a rhetorical analysis prompt:

rhetorical analysis ap lang example essay

From   CollegeBoard AP Lang 2017 Exam, FRQ Question 2

T: Luce uses many rhetorical strategies including  pathos *,*  antithesis , and a  humorous tone to soften up her audience before introducing her true reasons for being there. 

- Pathos Appeal

(“There is no audience more forgiving”)

- Antithesis

(“I am happy, I am less happy”)

- Humorous tone

(“consequently, no audience is more forgiving, I hope”)

BI: Today, just like for Luce, it is very difficult to give criticism to your peers.

Note: It is a good idea to make the TOBI about the size of your hand to make sure you don’t spend too much of your precious essay writing time on it.

What if I can't find any rhetorical devices that I recognize?

You can always go back and rely on tone as every piece of literature has one, even if it is just informative. If you know what they are doing, but not the name of the term, you can still just describe it and get the points. Additionally, make sure that you are familiar with  all the rhetorical devices that are a part of AP Lang!

🎥 Watch: AP Language -   Reading with an Analytical Mind

If it’s not an argumentative essay, what do you put in your thesis?

You state the most important writing choices the author made in order to impact the audience of the work.

Other Tips and Tricks

  • The big idea should show how this prompt applies to today. This will help you write your conclusion. In most language arts classes they teach you to simply restate your points, but not in AP Lang!
  • The first thing you are going to want to do is carefully read through and highlight any strategies you see.
  • Even if TOBI doesn’t work for you, it is a good idea to outline the essay. Even though it takes time, it will end up saving you time in the end because it gives you direction.
  • One of the most useful tools for the introduction is something called Soapstones. In this intro you are introducing the  S peaker,  O ccasion,  A udience,  P urpose,  S ubject,  T one, and  S tyle. (Keep in mind: You do not need to include EVERY ONE). But, most successful essays include a few of them.
  • DON’T SKIM! It will only hurt you in the long run, even if you think it might be saving you time.

rhetorical analysis ap lang example essay

Rhetorical Analysis Example Essay Prompt

The speech below was given at the site of the battle of Gettysburg by president Abraham Lincoln. Lincoln went on to describe his desire to save the union. Read the passage carefully and then in a well-developed essay, analyze the writing choices Lincoln makes to share his message with others. Support your analysis of his rhetoric with specific references from the text.

“Four score and seven years ago our fathers brought forth on this continent a new nation, conceived in Liberty and dedicated to the proposition that all men are created equal. Now we are engaged in a great civil war, testing whether that nation or any nation so conceived and so dedicated, can long endure. We are met on a great battle-field of that war. We have come to dedicate a portion of that field, as a final resting place for those who here gave their lives that that nation might live. It is altogether fitting and proper that we should do this. But, in a larger sense, we can not dedicate—we can not consecrate—we can not hallow—this ground. The brave men, living and dead, who struggled here, have  consecrated it, far above our poor power to add or detract. The world will little note, nor long remember what we say here, but it can never forget what they did here. It is for us the living, rather, to be dedicated here to the unfinished work which they who fought here have thus far so nobly advanced. It is rather for us to be here dedicated to the great task remaining before us—that from these honored dead we take increased devotion to that cause for which they gave the last full measure of devotion—that we here highly resolve that these dead shall not have died in vain—that this nation, under God, shall have a new birth of freedom—and that government of the people, by the people, for the people, shall not perish from the earth.”

Key Terms to Review ( 20 )

Fiveable

Stay Connected

© 2024 Fiveable Inc. All rights reserved.

AP® and SAT® are trademarks registered by the College Board, which is not affiliated with, and does not endorse this website.

AP English Language and Composition: Sample Rhetorical Analysis and Synthesis Questions

April 9, 2024.

AP English Language Sample Rhetorical Analysis and Synthesis Questions

The Rhetorical Analysis and Synthesis Essays are two of the three essays you’ll need to write as part of the AP English Language and Composition Exam . Read on for a sample of each, as well as tips for how to answer them. 

AP English Language and Composition: Sample Rhetorical Analysis Question

Read the following passage published back in 1967 by The New York Times. Then write an essay in which you analyze how the structure of the passage and the use of language help convey the writer’s views.

Sample Question Instructions:

  • Respond to the prompt with a thesis that may establish a line of reasoning.
  • Select and use evidence to develop and support the line of reasoning.
  • Explain the relationship between the evidence and the thesis.
  • Demonstrate an understanding of the rhetorical situation.
  • Use appropriate grammar and punctuation in communicating the argument.

Americans and Western Europeans, in their sensitivity to lingering problems around them, tend to make science and progress their scapegoats. There is a belief that progress has precipitated widespread unhappiness, anxieties, and other social and emotional problems. Science is viewed as a cold mechanical discipline having nothing to do with human warmth and the human spirit. 

But to many of us from the nonscientific East, science does not have such repugnant associations. We are not afraid of it, nor are we disappointed by it. We know all too painfully that our social and emotional problems festered long before the age of technology. To us, science is warm and reassuring. It promises hope. It is helping us at long last gain some control over our persecutory environments, alleviating age-old problems—not only physical but also, and especially, problems of the spirit.

Shiraz, for example, a city in southern Iran, has long been renowned for its rose gardens and nightingales; its poets, Sadi and Hafiz; and its mystical, ascetic philosophy, Sufism. Much poetry has been written in glorification of the spiritual attributes of this oasis city. And to be sure, Shiraz is a green, picturesque town, with a quaint bazaar and refreshing gardens. But in this “romantic” city thousands of emotionally disturbed and mentally retarded men, women, and children were, until recently, kept in chains in stifling prison cells and lunatic asylums. 

Every now and again, some were dragged, screaming and pleading, to a courtyard and flogged for not behaving “normally.” But for the most part, they were made to sit against damp walls, their hands and feet locked in chains, and thus immobilized, without even a modicum of affection from their helpless families and friends, they sat for weeks and months and years—often all their lives. Pictures of these wretched men, women, and children can still be seen in this “city of poetry,” this “city with a spiritual way of life.” 

It was only recently that a wealthy young Shirazi who, against the admonitions of his family, had studied psychology at the University of Tehran and foreign universities, returned to Shiraz and after considerable struggle with city officials succeeded in opening a psychiatric clinic, the first in those regions. After still more struggle, he arranged to have the emotionally disturbed and the mentally retarded transferred from prison to their homes, to hospitals, and to his clinic, where he and his staff now attend them. 

They are fortunate. All over Asia and other backward areas, emotionally disturbed men and women are still incarcerated in these medieval dungeons called lunatic asylums. The cruel rejection and punishment are intended to teach them a lesson or help exorcise evil spirits. 

The West, still bogged down in its ridiculous romanticism, would like to believe that emotional disturbances, dope addiction, delinquency are all modern problems brought on by technological progress, and that backward societies are too spiritual and beautiful to need the ministrations of science. But while the West can perhaps afford to think this way, the people of backward lands cannot. . . . 

. . .The obstacles are awesome, the inertia too entrenched, the people’s suffering too anguished, their impatience too eruptive. Moreover, the total cultural reorganizations such as Asia and Africa are undergoing inevitably engender their own temporary dislocations and confusions. But their goals, the direction, remain constant. We are on the move, however awkwardly at first, to a saner, better world.

How to Answer the AP English Language and Composition Rhetorical Analysis Question

Go back to the original question, which asks you to analyze two features of the passage: (1) its structure, or organization, and (2) its language. The first aspect is fairly specific. As you read the passage, you need to observe what the author discusses first, second, third, and so on. Your essay should explain not only the order of ideas but the reasons the author may have chosen that order. 

The second part of the question is more general. It invites you to analyze the use of language, which may include the author’s choice of words (diction), syntax (word order), figures of speech, use of evidence (such as statistics or logical reasoning), sentence structure, rhythm, sound, tone, or just about any other characteristics of style and rhetoric you choose. 

Although the question directs you to write about two different aspects of the passage, the essay itself should be unified. That is, a good essay should not consist of, say, two disparate paragraphs, one exclusively devoted to structure and another to language. Rather, the essay should include material that shows the interrelationship of structure and language in the passage and how those elements contribute to the meaning and effect of the passage. This might be covered in a separate paragraph, or it could be woven into the overall fabric of the essay. 

Before you begin to write, read the passage at least twice: once for an overview and once as you write your analysis. You may notice early on that the opening paragraph contains generalizations about Westerners’ concepts of science and progress. Then the author contrasts the Western view of science and progress with the Eastern view. Immediately, you see that the author, by using the first-person pronoun (as in “many of us”) is speaking from the perspective of an Easterner. Consequently, his discussion of Eastern views is apt to come across as more well-informed, more authoritative, perhaps more personal. 

To support his position, the author gives an extended example—the city of Shiraz—to illustrate just how different the East is from the West. The description and vivid images of Shiraz memorably convey the idea that the “spiritual way of life” has a side to it that many Westerners don’t know about. This is the heart of the passage. The use of quotation marks around “romantic” and “city of poetry” is meant to point out the discrepancy between the idealized and real versions of Shiraz. 

Nearing the end, the author reiterates his initial contrast between West and East, with emphasis on the East. The last paragraph offers a generalized statement about conditions in Asia and Africa, reminding the reader of the contrast made at the very beginning of the passage. Tying the end to the beginning of the passage creates a sense of unity—a desirable feature in any piece of writing.

AP English Language and Composition: Sample Argument Question

The following paragraph is adapted from Mirror for Man, a book written by anthropologist Clyde Kluckhorn in the middle of the twentieth century. Read the passage carefully. Then, write an essay that examines the extent to which the author’s characterization of the United States holds true today. Use appropriate evidence to support your argument. 

Sample Question Instructions: 

  • Respond to the prompt with a thesis that may establish a line of reasoning. 
  • Select and use evidence to develop and support the line of reasoning. 
  • Explain the relationship between the evidence and the thesis. 
  • Demonstrate an understanding of the rhetorical situation. 

Technology is valued as the very basis of the capitalistic system. Possession of gadgets is esteemed as a mark of success to the extent that persons are judged not by the integrity of their characters or by the originality of their minds but by what they seem to be—so far as can be measured by their wealth or by the variety and material goods which they display. “Success” is measured by their investments, homes, and lifestyles— not by their number of mistresses as in some cultures.

How to Answer the AP English Language and Composition Argument Question

Whether you agree, disagree, or have mixed views on the content of the passage, your job is to write a convincing argument that expresses your opinion. Initially, the word argument may suggest conflict or confrontation. But rest assured that your essay need not be combative. Rather, make it a calmly-reasoned explanation of your opinion on a debatable subject. Your goal is to persuade the reader that your opinion, supported by examples, facts, and other appropriate evidence, is correct. 

If you have strong feelings about the topic, of course you should state them in your essay. But express them in calm, rational language. Be mindful that the essay should not be an emotional rant for or against the issue. 

Consider first whether you agree with Kluckhorn’s definition of “success.” Is it, as Kluckhorn asserts, measured by income and material possessions? Or do you think that a more accurate standard of success in today’s America should be determined by less tangible criteria—things such as happiness or self-respect? Or do you stand somewhere in between those two extremes? 

The actual position you take on the issue is less crucial than your ability to support it fully by drawing from your knowledge, background, experience, or observation. Regardless of your position, be sure to include more than one example. An argument that relies on a single example, however compelling, will fall flat. 

In the prompt, Kluckhorn’s notion of success seems to refer broadly to American society. Resist responding in kind. That is, a short essay shouldn’t focus on the whole of society but only on an identifiable segment—perhaps college-educated professionals or urban, blue- collar Americans. The point is that a narrowly focused essay on a limited topic will always turn out better than one that tries to cover too much ground in just a few paragraphs.

AP Biology Resources

  • About the AP Biology Exam
  • Top AP Biology Exam Strategies
  • Top 5 Study Topics and Tips for the AP Biology Exam
  • AP Biology Short Free-Response Questions
  • AP Biology Long Free-Response Questions

AP Psychology Resources

  • What’s Tested on the AP Psychology Exam?
  • Top 5 Study Tips for the AP Psychology Exam
  • AP Psychology Key Terms
  • Top AP Psychology Exam Multiple-Choice Question Tips
  • Top AP Psychology Exam Free Response Questions Tips
  • AP Psychology Sample Free Response Question

AP English Language and Composition Resources

  • What’s Tested on the AP English Language and Composition Exam?
  • Top 5 Tips for the AP English Language and Composition Exam
  • Top Reading Techniques for the AP English Language and Composition Exam
  • How to Answer the AP English Language and Composition Essay Questions 
  • AP English Language and Composition Exam Sample Essay Questions
  • AP English Language and Composition Exam Multiple-Choice Questions

AP Human Geography Resources

  • What’s Tested On the AP Human Geography Exam?
  • AP Human Geography FAQs
  • AP Human Geography Question Types and Strategies
  • Top 5 Study Tips for the AP Human Geography Exam

FOLLOW ALONG ON SOCIAL

logo-type-white

AP® English Language

3 ap® english language rhetorical essay strategies.

  • The Albert Team
  • Last Updated On: March 1, 2022

3_ap_english_language rhetorical essay strategies

The AP® English Language rhetorical essay can be nightmare inducing for some AP® students, but there is no need for fear. In this exam review we will lay out helpful strategies to get you through the rhetorical essays in no time.

Rhetorical Strategy #1: Dissecting the Prompt

The first rhetorical essay strategy is to dissect the prompt. Understanding what the rhetorical essay wants from you is essential. It is important for you to read the prompt carefully for every essay, but critical reading is even more essential to the rhetorical essay. Your rhetorical prompt that you will be given for the AP® English Language exam will contain two elements. The first element is the concrete task that the prompt is asking of you, which is always to analyze the passage that follows. The second part of the prompt is a more abstract task that is not directly asked for in the prompt, but it is implied. By completely understanding both parts of the prompt, you will be able to give a complete essay that will get you to a higher score.

One example of a prompt from an AP® English Language rhetorical essay is this one from the 2008 exam . The prompt reads:

“In the following passage from The Great Influenza, an account of the 1918 flu epidemic, author John M. Barry writes about scientists and their research. Read the passage carefully. Then, in a well-written essay, analyze how Barry uses rhetorical strategies.”

Here you can see the concrete task that the examiners are asking. They want you to analyze the passage for rhetorical strategies; however, you must figure out what you are analyzing the passage for. That is the more abstract concept that you need to dissect the prompt to find. In the case of Barry’s passage you will need to analyze how he uses rhetorical strategies in order to portray scientific research. We know this, because if you look at the prompt, it specifically states what Barry did in his work, which was to write about science and research. That is your abstract task.

Once you have found your concrete task and your abstract task, a great strategy is to write it down to keep you focused throughout your essay . Using the example above this would look like the following:

Analyze how Barry uses rhetorical strategies in order to portray scientific research.

That sentence is what you must follow when writing your essay, and if you successfully keep to this task, then you will move closer to that high score.

Rhetorical Essay Strategy #2: Stick to the Format

This next rhetorical essay strategy is the key to great organization and structure that will put your test anxiety to bed. There is a simple paragraph structure for the body paragraphs of the AP® English Language rhetorical essay that will allow you to think, write, and score higher, faster. You need to begin each body paragraph with an assertion or claim. That is the point that you are trying to make clear to your audience what you will be proving. A great example of this is from the 2006 AP® English Language rhetorical essay. Below is student 2B’s opening sentence for her first body paragraph.

“The diction of the passage fully relays Hazlitt’s position about money ( student 2B ).”

You can see how the student directly asserts what he or she will be proving in this statement. The next step in constructing your body paragraph is to give one to two pieces of textual evidence. Be sure to state why these quotations relate back to your claim, otherwise they will be deemed irrelevant by the examiners. An example of this is the next sentence in student 2B’s body paragraph about diction. Here, the student brings in elements from the text to support his or her claim about Hazlitt using diction.

“’Rejected’, ‘contempt’, ‘disparaged’, ‘scrutinized’, ‘irksome’, ‘deprived’, ‘assailed’, ‘chagrin’; the endless repetition of such discouragement shows just how emphatically Hazlitt money is requisite for happy life (student 2B).”

The final part of this strategy for conquering the body paragraphs of your rhetorical essays is to end those body paragraphs with a thorough analysis. This is the aspect of the exam where you can put your way of looking at the text into your essay.

An example of this is at the end of student 2B’s body paragraph where he or she states, “The irony of the last sentences is negative, conveying the utter hopelessness of one without money. Though one may have none in life, pitiless men will continue to mock one’s circumstances even after death! (student 2B)”

This analysis of the text adds to the textual examples above and continues to bring in new logic from the student.

When this format of a body paragraph is followed, then it is extremely effective. The essay becomes clear, assertive, and easy to follow for the examiners. Follow this rhetorical essay strategy and you are even closer to getting that 5 on the exam.

Rhetorical Essay Strategy #3: LORA

As you are looking at your AP® English Language rhetorical essay prompt and passage it is important to remember the mnemonic device, LORA. LORA stands for Language, Organization, and Rhetorical Appeals. These elements will help you form your argument.

When you read through your passage you want to think about how the author is utilizing language. Is he or she using figurative language effectively? Is there imagery within the passage? Does the diction of the passage make it more rhetorically persuasive? You should not use all of these, but picking one and analyzing it clearly in one paragraph will keep you focused on how the author uses rhetoric, which is the main task of this essay.

An example of this was in the 2006 AP® English Language rhetorical essay. Student 2A begins his or her first body paragraph with, “One of Hazlitt’s most effective methods of promoting the importance of money is his strong diction (student 2A).” This student begins his or her essay with focusing on diction as how the language is used. He or she then goes on to explain why diction betters Hazlitt’s argument, which is exactly what you must do for your own rhetorical essay.

The organization of the author is the next part of your answer to the prompt. You want to look at how the author organized his or her ideas within the passage to support his or her own argument. By pointing out the organization, or structure, of the work and how it adds to the overall persuasiveness, you will bring two of the three most important elements of rhetoric together in your essay.

After organization you need to look at the rhetoric appeals. You may know them by the names logos, pathos, and ethos. It is suggested that you cover as many of these as possible; however, if time does not permit or if the passage uses one more than the other, then you should focus on one appeal.

One example of using pathos in an essay is from student 2A from the 2006 prompt. “Hazlitt plays on the audience’s heartstrings for more than enough time to convince them of the importance of having money (student 2A).” While it would have been better for the student to directly say that this is pathos, he or she does thoroughly explain the appeal to the passions, or pathos.

Key Takeaways

When taking the AP® English Language rhetoric essay you just need to remember these three rhetorical essay strategies: dissect the prompt, follow the format, and always include LORA. If you can follow them, then you are already on your way to a 5 on the AP® English Language exam .

Let’s put everything into practice. Try this AP® English Language practice question:

Rhetorical Considerations AP® English Language Practice Question

Looking for more AP® English Language practice?

Check out our other articles on AP® English Language .

You can also find thousands of practice questions on Albert.io. Albert.io lets you customize your learning experience to target practice where you need the most help. We’ll give you challenging practice questions to help you achieve mastery of AP® English Language.

Start practicing here .

Are you a teacher or administrator interested in boosting AP® English Language student outcomes?

Learn more about our school licenses here .

Interested in a school license?​

Popular posts.

AP® Physics I score calculator

AP® Score Calculators

Simulate how different MCQ and FRQ scores translate into AP® scores

rhetorical analysis ap lang example essay

AP® Review Guides

The ultimate review guides for AP® subjects to help you plan and structure your prep.

rhetorical analysis ap lang example essay

Core Subject Review Guides

Review the most important topics in Physics and Algebra 1 .

rhetorical analysis ap lang example essay

SAT® Score Calculator

See how scores on each section impacts your overall SAT® score

rhetorical analysis ap lang example essay

ACT® Score Calculator

See how scores on each section impacts your overall ACT® score

rhetorical analysis ap lang example essay

Grammar Review Hub

Comprehensive review of grammar skills

rhetorical analysis ap lang example essay

AP® Posters

Download updated posters summarizing the main topics and structure for each AP® exam.

Global Health and Education

2020 ap english exam: how to write a rhetorical analysis essay.

In light of The College Board recently announcing that this year’s AP English Language and Composition exam will be only one question , a 45-minute rhetorical analysis essay,  it would probably be a good idea to freshen up on your essay-writing skills. The benefits of doing this won’t stop once high-school ends- knowing how to write rhetorical analysis essays will also be extremely helpful for future college and professional endeavors where you might be required to examine a text or prepare a report on one. In this article you will learn the breakdown of every section of the rhetorical analysis essay, and what you should be including in order to earn a five this May.

2020 AP English Language and Composition Exam Changes: How to write a 45-minute rhetorical analysis essay?

Introduction.

The first part of the rhetorical essay is the introduction. The introduction can be broken down into four simple parts, and because of this, if you feel that you are running short on time, don’t be afraid to actually wait until the end to write your introduction. This could possibly be beneficial since you’ll know exactly where the rest of your essay is going.

The beginning of your first paragraph will include naming the piece you are analyzing, then contextualizing it and including any relevant background information. This can include historical events that were taking place during the writing of this essay (for example, World War II, the Civil Rights Movement, a certain political campaign, etc.), the time period, the country, conflict, and anything else that could serve as the something affecting the purpose or exigence for the piece.

The last portion of the first paragraph is the thesis. The thesis emphasizes the purpose of the writing- the why . Why is the author doing any of this? What’s the reason? Your thesis is basically the entire argument of your essay summed up into one sentence. Another thing that’s important to add in your thesis is the effectiveness of the author’s execution of their argument. Did they do a good job of getting their point across? Make sure to have evidence to back this point up later.

Body Paragraphs

Now you have to analyze the rhetorical device that you mentioned in your topic sentence. To do this, you should start by quoting passages from your text that show usage of the device, then elaborate on why that’s true and how it affects the delivery of the argument, as well as the argument itself. The analysis section will be the largest part of each body paragraph. Always be thinking about the argument that your author is making while doing your analysis. 

Contradiction

Now that you’ve spent so much time examining the author’s argument in tiny pieces, it’s time to bring it all together for the reader. Go back through the main points made throughout the essay and how they showcase the main points of the author. Don’t be afraid to re-mention how the rhetorical choices strengthen said main points.

Works Cited

“Examples of Rhetorical Devices.” Your Dictionary. Accessed April 21, 2020. https://examples.yourdictionary.com/examples-of-rhetorical-devices.html .

© Copyright 2020 Moosmosis – All rights reserved

rhetorical analysis ap lang example essay

Please Like our Facebook page to support! 🙂

Share this:.

Tagged as: AP English , ap literature , english , learning , Literature , literature essay , literature high school , school , writing

Published by Elle Ward

Hi! I'm an aspiring journalist and writer. A picture may be worth a thousand words, but a novel can create a thousand pictures. View all posts by Elle Ward

7 replies »

Like Liked by 1 person

Leave a comment Cancel reply

Thank you for visiting, today’s top posts, our community, choose a language.

Please Like to Support!

Have a language expert improve your writing

Run a free plagiarism check in 10 minutes, generate accurate citations for free.

  • Knowledge Base
  • How to write a rhetorical analysis | Key concepts & examples

How to Write a Rhetorical Analysis | Key Concepts & Examples

Published on August 28, 2020 by Jack Caulfield . Revised on July 23, 2023.

A rhetorical analysis is a type of essay  that looks at a text in terms of rhetoric. This means it is less concerned with what the author is saying than with how they say it: their goals, techniques, and appeals to the audience.

Instantly correct all language mistakes in your text

Upload your document to correct all your mistakes in minutes

upload-your-document-ai-proofreader

Table of contents

Key concepts in rhetoric, analyzing the text, introducing your rhetorical analysis, the body: doing the analysis, concluding a rhetorical analysis, other interesting articles, frequently asked questions about rhetorical analysis.

Rhetoric, the art of effective speaking and writing, is a subject that trains you to look at texts, arguments and speeches in terms of how they are designed to persuade the audience. This section introduces a few of the key concepts of this field.

Appeals: Logos, ethos, pathos

Appeals are how the author convinces their audience. Three central appeals are discussed in rhetoric, established by the philosopher Aristotle and sometimes called the rhetorical triangle: logos, ethos, and pathos.

Logos , or the logical appeal, refers to the use of reasoned argument to persuade. This is the dominant approach in academic writing , where arguments are built up using reasoning and evidence.

Ethos , or the ethical appeal, involves the author presenting themselves as an authority on their subject. For example, someone making a moral argument might highlight their own morally admirable behavior; someone speaking about a technical subject might present themselves as an expert by mentioning their qualifications.

Pathos , or the pathetic appeal, evokes the audience’s emotions. This might involve speaking in a passionate way, employing vivid imagery, or trying to provoke anger, sympathy, or any other emotional response in the audience.

These three appeals are all treated as integral parts of rhetoric, and a given author may combine all three of them to convince their audience.

Text and context

In rhetoric, a text is not necessarily a piece of writing (though it may be this). A text is whatever piece of communication you are analyzing. This could be, for example, a speech, an advertisement, or a satirical image.

In these cases, your analysis would focus on more than just language—you might look at visual or sonic elements of the text too.

The context is everything surrounding the text: Who is the author (or speaker, designer, etc.)? Who is their (intended or actual) audience? When and where was the text produced, and for what purpose?

Looking at the context can help to inform your rhetorical analysis. For example, Martin Luther King, Jr.’s “I Have a Dream” speech has universal power, but the context of the civil rights movement is an important part of understanding why.

Claims, supports, and warrants

A piece of rhetoric is always making some sort of argument, whether it’s a very clearly defined and logical one (e.g. in a philosophy essay) or one that the reader has to infer (e.g. in a satirical article). These arguments are built up with claims, supports, and warrants.

A claim is the fact or idea the author wants to convince the reader of. An argument might center on a single claim, or be built up out of many. Claims are usually explicitly stated, but they may also just be implied in some kinds of text.

The author uses supports to back up each claim they make. These might range from hard evidence to emotional appeals—anything that is used to convince the reader to accept a claim.

The warrant is the logic or assumption that connects a support with a claim. Outside of quite formal argumentation, the warrant is often unstated—the author assumes their audience will understand the connection without it. But that doesn’t mean you can’t still explore the implicit warrant in these cases.

For example, look at the following statement:

We can see a claim and a support here, but the warrant is implicit. Here, the warrant is the assumption that more likeable candidates would have inspired greater turnout. We might be more or less convinced by the argument depending on whether we think this is a fair assumption.

Prevent plagiarism. Run a free check.

Rhetorical analysis isn’t a matter of choosing concepts in advance and applying them to a text. Instead, it starts with looking at the text in detail and asking the appropriate questions about how it works:

  • What is the author’s purpose?
  • Do they focus closely on their key claims, or do they discuss various topics?
  • What tone do they take—angry or sympathetic? Personal or authoritative? Formal or informal?
  • Who seems to be the intended audience? Is this audience likely to be successfully reached and convinced?
  • What kinds of evidence are presented?

By asking these questions, you’ll discover the various rhetorical devices the text uses. Don’t feel that you have to cram in every rhetorical term you know—focus on those that are most important to the text.

The following sections show how to write the different parts of a rhetorical analysis.

Like all essays, a rhetorical analysis begins with an introduction . The introduction tells readers what text you’ll be discussing, provides relevant background information, and presents your thesis statement .

Hover over different parts of the example below to see how an introduction works.

Martin Luther King, Jr.’s “I Have a Dream” speech is widely regarded as one of the most important pieces of oratory in American history. Delivered in 1963 to thousands of civil rights activists outside the Lincoln Memorial in Washington, D.C., the speech has come to symbolize the spirit of the civil rights movement and even to function as a major part of the American national myth. This rhetorical analysis argues that King’s assumption of the prophetic voice, amplified by the historic size of his audience, creates a powerful sense of ethos that has retained its inspirational power over the years.

The body of your rhetorical analysis is where you’ll tackle the text directly. It’s often divided into three paragraphs, although it may be more in a longer essay.

Each paragraph should focus on a different element of the text, and they should all contribute to your overall argument for your thesis statement.

Hover over the example to explore how a typical body paragraph is constructed.

King’s speech is infused with prophetic language throughout. Even before the famous “dream” part of the speech, King’s language consistently strikes a prophetic tone. He refers to the Lincoln Memorial as a “hallowed spot” and speaks of rising “from the dark and desolate valley of segregation” to “make justice a reality for all of God’s children.” The assumption of this prophetic voice constitutes the text’s strongest ethical appeal; after linking himself with political figures like Lincoln and the Founding Fathers, King’s ethos adopts a distinctly religious tone, recalling Biblical prophets and preachers of change from across history. This adds significant force to his words; standing before an audience of hundreds of thousands, he states not just what the future should be, but what it will be: “The whirlwinds of revolt will continue to shake the foundations of our nation until the bright day of justice emerges.” This warning is almost apocalyptic in tone, though it concludes with the positive image of the “bright day of justice.” The power of King’s rhetoric thus stems not only from the pathos of his vision of a brighter future, but from the ethos of the prophetic voice he adopts in expressing this vision.

The conclusion of a rhetorical analysis wraps up the essay by restating the main argument and showing how it has been developed by your analysis. It may also try to link the text, and your analysis of it, with broader concerns.

Explore the example below to get a sense of the conclusion.

It is clear from this analysis that the effectiveness of King’s rhetoric stems less from the pathetic appeal of his utopian “dream” than it does from the ethos he carefully constructs to give force to his statements. By framing contemporary upheavals as part of a prophecy whose fulfillment will result in the better future he imagines, King ensures not only the effectiveness of his words in the moment but their continuing resonance today. Even if we have not yet achieved King’s dream, we cannot deny the role his words played in setting us on the path toward it.

If you want to know more about AI tools , college essays , or fallacies make sure to check out some of our other articles with explanations and examples or go directly to our tools!

  • Ad hominem fallacy
  • Post hoc fallacy
  • Appeal to authority fallacy
  • False cause fallacy
  • Sunk cost fallacy

College essays

  • Choosing Essay Topic
  • Write a College Essay
  • Write a Diversity Essay
  • College Essay Format & Structure
  • Comparing and Contrasting in an Essay

 (AI) Tools

  • Grammar Checker
  • Paraphrasing Tool
  • Text Summarizer
  • AI Detector
  • Plagiarism Checker
  • Citation Generator

The goal of a rhetorical analysis is to explain the effect a piece of writing or oratory has on its audience, how successful it is, and the devices and appeals it uses to achieve its goals.

Unlike a standard argumentative essay , it’s less about taking a position on the arguments presented, and more about exploring how they are constructed.

The term “text” in a rhetorical analysis essay refers to whatever object you’re analyzing. It’s frequently a piece of writing or a speech, but it doesn’t have to be. For example, you could also treat an advertisement or political cartoon as a text.

Logos appeals to the audience’s reason, building up logical arguments . Ethos appeals to the speaker’s status or authority, making the audience more likely to trust them. Pathos appeals to the emotions, trying to make the audience feel angry or sympathetic, for example.

Collectively, these three appeals are sometimes called the rhetorical triangle . They are central to rhetorical analysis , though a piece of rhetoric might not necessarily use all of them.

In rhetorical analysis , a claim is something the author wants the audience to believe. A support is the evidence or appeal they use to convince the reader to believe the claim. A warrant is the (often implicit) assumption that links the support with the claim.

Cite this Scribbr article

If you want to cite this source, you can copy and paste the citation or click the “Cite this Scribbr article” button to automatically add the citation to our free Citation Generator.

Caulfield, J. (2023, July 23). How to Write a Rhetorical Analysis | Key Concepts & Examples. Scribbr. Retrieved June 20, 2024, from https://www.scribbr.com/academic-essay/rhetorical-analysis/

Is this article helpful?

Jack Caulfield

Jack Caulfield

Other students also liked, how to write an argumentative essay | examples & tips, how to write a literary analysis essay | a step-by-step guide, comparing and contrasting in an essay | tips & examples, "i thought ai proofreading was useless but..".

I've been using Scribbr for years now and I know it's a service that won't disappoint. It does a good job spotting mistakes”

  • Our Writers
  • How to Order
  • Assignment Writing Service
  • Report Writing Service
  • Buy Coursework
  • Dissertation Writing Service
  • Research Paper Writing Service
  • All Essay Services
  • Buy Research Paper
  • Buy Term Paper
  • Buy Dissertation
  • Buy Case study
  • Buy Presentation
  • Buy Personal statement

User Icon

Rhetorical Analysis Essay

Rhetorical Analysis Essay Example

Nova A.

Rhetorical Analysis Essay Example - Free Samples

11 min read

Rhetorical Analysis Essay Example

People also read

Rhetorical Analysis Essay - A Complete Guide With Examples

320+ Best Rhetorical Analysis Essay Topics

Crafting an Effective Rhetorical Analysis Essay Outline - Free Samples!

Ethos, Pathos, and Logos - Structure, Usage & Examples

Writing a rhetorical analysis essay for academics can be really demanding for students. This type of paper requires high-level analyzing abilities and professional writing skills to be drafted effectively.

As this essay persuades the audience, it is essential to know how to take a strong stance and develop a thesis. 

This article will find some examples that will help you with your rhetorical analysis essay writing effortlessly. 

Arrow Down

  • 1. Good Rhetorical Analysis Essay Example
  • 2. Rhetorical Analysis Essay Example AP Lang 2023
  • 3. Rhetorical Analysis Essay Examples for Students 
  • 4. Writing a Visual Rhetorical Analysis Essay with Example 
  • 5. Rhetorical Analysis Essay Writing Tips

Good Rhetorical Analysis Essay Example

The step-by-step writing process of a rhetorical analysis essay is far more complicated than ordinary academic essays. This essay type critically analyzes the rhetorical means used to persuade the audience and their efficiency. 

The example provided below is the best rhetorical analysis essay example:

Rhetorical Analysis Essay Sample

In this essay type, the author uses rhetorical approaches such as ethos, pathos, and logos .  These approaches are then studied and analyzed deeply by the essay writers to weigh their effectiveness in delivering the message.

Let’s take a look at the following example to get a better idea;

Title: Analyzing the Rhetoric of John F. Kennedy's Inaugural Address

Introduction:

Rhetoric is a powerful tool leaders use to persuade and inspire their audiences. One of the most iconic speeches in American history is John F. Kennedy's Inaugural Address, delivered on January 20, 1961. 

In this rhetorical analysis essay, we will dissect Kennedy's speech to understand how he effectively employed rhetorical techniques to convey his vision for the United States and its role in the world. By examining Kennedy's use of ethos, pathos, and logos, as well as his adept use of rhetorical devices, we can gain insight into the impact of his words on the American people and the world at large.

Body:

Ethos: Establishing Credibility

John F. Kennedy, as the newly elected President of the United States, begins his address by establishing his credibility (ethos) through references to the Founding Fathers and the Constitution. 

He aligns himself with the principles of the nation's founders, positioning himself as a guardian of American values. By doing so, he inspires confidence in his leadership and fosters a sense of unity among his diverse audience.

Pathos: Evoking Emotion

Throughout his address, Kennedy masterfully employs emotional appeals (pathos) to engage and connect with his audience. He speaks of the "long twilight struggle" and the "bond that spans the seas," invoking a sense of shared responsibility and destiny among Americans. 

His call to "ask not what your country can do for you – ask what you can do for your country" resonates deeply, stirring feelings of patriotism and duty. Kennedy's use of emotionally charged phrases fosters a sense of national purpose.

Logos: Using Logic and Reasoning

Kennedy integrated logical arguments (logos) into his speech by addressing the global challenges of the Cold War era. He speaks of the need to "avoid the disaster of a nuclear war" and emphasizes the importance of diplomacy and cooperation with other nations. 

By presenting these arguments, he appeals to the audience's sense of reason and pragmatism, outlining a clear and rational path forward for the nation.

Rhetorical Devices: Amplifying Impact

Kennedy employs several rhetorical devices to amplify the impact of his address. Anaphora, the repetition of the phrase "Let both sides" in the context of nuclear disarmament, emphasizes the urgency of the situation and the need for cooperation. 

Antithesis, as seen in "We shall pay any price, bear any burden," creates a stark contrast that underscores the sacrifices required for freedom. These rhetorical devices enhance the overall persuasiveness and memorability of his speech.

Conclusion:

John F. Kennedy's Inaugural Address is a testament to the power of rhetoric in shaping public opinion and inspiring action. By skillfully weaving ethos, pathos, and logos into his speech and using rhetorical devices to amplify his message, Kennedy effectively communicated his vision for America's role in the world. His call for unity, service, and global cooperation resonated with the American people and left an enduring impact. 

As we analyze his rhetoric, we are reminded of the enduring relevance of persuasive communication in leadership and the ability of a well-crafted speech to mobilize a nation toward a common goal. Kennedy's inaugural address serves as a timeless example of the art of persuasion at its finest.

The outline and structure of a rhetorical analysis essay are important. 

According to the essay outline, the essay is divided into three sections: 

  • Introduction
  • Ethos 
  • Logos 

A rhetorical analysis essay outline is the same as the traditional one. The different parts of the rhetorical analysis essay are written in the following way:

Rhetorical Analysis Introduction Example

The introductory paragraph of a rhetorical analysis essay is written for the following purpose:

  • To provide basic background information about the chosen author and the text.
  • Identify the target audience of the essay. 

An introduction for a rhetorical essay is drafted by:

  • Stating an opening sentence known as the hook statement. This catchy sentence is prepared to grab the audience’s attention to the paper. 
  • After the opening sentence, the background information of the author and the original text are provided. 

For example, a rhetorical analysis essay written by Lee Jennings on“The Right Stuff” by David Suzuki. Lee started the essay by providing the introduction in the following way:

David Suzuki’s “The Right Stuff” features a gracious, entertaining, and informative style. We associate with this well-known host of The Nature of Things. 

He begins with the interesting speculation from the book Is There Life After High School? that “impressions formed in high school are more vivid and indelible than those formed at any other time in life.

Analysis of the Example: 

  • Suzuki stresses the importance of high school education. He prepares his readers for a proposal to make that education as valuable as possible.
  • A rhetorical analysis can show how successful Suzuki was in using logos, pathos, and ethos. He had a strong ethos because of his reputation. 
  • He also used pathos to appeal to parents and educators. However, his use of logos could have been more successful.
  • Here Jennings stated the background information about the text and highlighted the rhetorical techniques used and their effectiveness. 

Thesis Statement Example for Rhetorical Analysis Essay 

A thesis statement of a rhetorical analysis essay is the writer’s stance on the original text. It is the argument that a writer holds and proves it using the evidence from the original text. 

A thesis statement for a rhetorical essay is written by analyzing the following elements of the original text:

  • Diction - It refers to the author’s choice of words and the tone
  • Imagery - The visual descriptive language that the author used in the content. 
  • Simile - The comparison of things and ideas

In Jennings's analysis of “The Right Stuff,” the thesis statement was:

"Suzuki has strong beliefs, but his argument is not convincing because it is not supported by evidence."

Example For Rhetorical Analysis Thesis Statement

Rhetorical Analysis Body Paragraph Example 

In the body paragraphs of your rhetorical analysis essay, you dissect the author's work, analyze their use of rhetorical techniques, and provide evidence to support your analysis. 

Let's look at an example that analyzes the use of ethos in David Suzuki's essay:

Body Paragraph - Analyzing Ethos

David Suzuki, a well-known host of "The Nature of Things" and a respected scientist in the field of environmentalism, expertly employs ethos in his essay, "The Right Stuff." Suzuki's ethos is immediately established through his impressive background and extensive experience in the field. As a scientist, he has dedicated his career to understanding and addressing environmental issues, which lends him considerable credibility on the subject.

Moreover, Suzuki's role as a popular television host further solidifies his ethos. His presence in the media has made him a recognizable and influential figure, particularly among those interested in environmental matters. When Suzuki speaks, his audience often associates his words with authority and expertise in the field.

Throughout his essay, Suzuki utilizes his ethos to build trust with his readers. He references his own experiences and interactions, stating, "In a lifetime of lecturing, I've discovered that people have strong opinions on environmental issues." By sharing his encounters, Suzuki humanizes himself, making his readers more inclined to trust his perspective.

Suzuki's ethos is a pivotal component of his argument. It strengthens his credibility and positions him as a reliable source of information on environmental issues. As a result, his call for greater environmental awareness and action carries significant weight and persuasiveness.

Rhetorical Analysis Conclusion Example

All the body paragraphs lead the audience towards the conclusion.

For example, the conclusion of “The Right Stuff” is written in the following way by Jennings:

The major question overlooked by Suzuki’s essay—one of logistics-- is how can schools, understaffed and overstressed, add the difficult subject of sex education to their curriculum. 

Admittedly, David Suzuki wrote his essay at a time when education budgets were in better shape than they are today. And he certainly makes an excellent point that educators should respect their students and appeal to their interests. 

Nevertheless, his argument for sex education in schools needs further thinking. Suzuki has a strong ethos and uses persuasive pathos well in this argument. 

However, he needs to use more logos to make his point clear. He can only hope to get people's attention. It is up to them whether or not they want to use his ideas in the schools.

In the conclusion section, Jennings summarized the major points and restated the thesis statement to prove them. 

Rhetorical Essay Example For The Right Stuff by David Suzuki

Rhetorical Analysis Essay Example AP Lang 2023

Writing a rhetorical analysis for the AP Language and Composition course can be challenging. So drafting it correctly is important to earn good grades. 

To make your essay effective and winning, follow the tips provided by professionals below:

Step #1: Understand the Prompt

Understanding the prompt is the first thing to produce an influential rhetorical paper. It is mandatory for this academic writing to read and understand the prompt to know what the task demands from you. 

Step #2: Stick to the Format

The content for the rhetorical analysis should be appropriately organized and structured. For this purpose, a proper outline is drafted. 

The rhetorical analysis essay outline divides all the information into different sections, such as the introduction, body, and conclusion.  The introduction should explicitly state the background information and the thesis statement. 

All the body paragraphs should start with a topic sentence to convey a claim to the readers. Provide a thorough analysis of these claims in the paragraph to support your topic sentence. 

Step #3: Use Rhetorical Elements to Form an Argument 

Analyze the following things in the text to form an argument for your essay:

  • Language (tone and words)
  • Organizational structure
  • Rhetorical Appeals ( ethos, pathos, and logos) 

Once you have analyzed the rhetorical appeals and other devices like imagery and diction, you can form a strong thesis statement. The thesis statement will be the foundation on which your essay will be standing. 

AP Language Rhetorical Essay Sample

AP Rhetorical Analysis Essay Template

Rhetorical Analysis Essay Example AP Lang

AP Lang Rhetorical Analysis Essay Example

Rhetorical Analysis Essay Examples for Students 

Here are a few more examples to help the students write a rhetorical analysis essay:

Rhetorical Analysis Essay Example Ethos, Pathos, Logos

Rhetorical Analysis Essay Example Outline

Rhetorical Analysis Essay Example College

Rhetorical Analysis Essay Example APA Format

Compare and Contrast Rhetorical Analysis Essay Example

Comparative Rhetorical Analysis Essay Example

How to Start Rhetorical Analysis Essay Example

Rhetorical Analysis Essay Example High School

Rhetorical Analysis Essay Example APA Sample

Rhetorical Analysis Essay Example Of a Song

Florence Kelley Speech Rhetorical Analysis Essay Example

Rhetorical Analysis Essay Example MLA

Writing a Visual Rhetorical Analysis Essay with Example 

The visual rhetorical analysis essay determines how pictures and images communicate messages and persuade the audience. 

Usually, visual rhetorical analysis papers are written for advertisements. This is because they use strong images to convince the audience to behave in a certain way. 

To draft a perfect visual rhetorical analysis essay, follow the tips below:

  • Analyze the advertisement deeply and note every minor detail. 
  • Notice objects and colors used in the image to gather every detail.
  • Determine the importance of the colors and objects and analyze why the advertiser chose the particular picture. 
  • See what you feel about the image.
  • Consider the objective of the image. Identify the message that the image is portraying. 
  • Identify the targeted audience and how they respond to the picture. 

An example is provided below to give students a better idea of the concept. 

Simplicity Breeds Clarity Visual Rhetorical Analysis Essay Example

Rhetorical Analysis Essay Writing Tips

Follow the tips provided below to make your rhetorical writing compelling. 

  • Choose an engaging topic for your essay. The rhetorical analysis essay topic should be engaging to grab the reader’s attention.
  • Thoroughly read the original text.
  • Identify the SOAPSTone. From the text, determine the speaker, occasions, audience, purpose, subject, and tone.
  • Develop a thesis statement to state your claim over the text.
  • Draft a rhetorical analysis essay outline.
  • Write an engaging essay introduction by giving a hook statement and background information. At the end of the introductory paragraph, state the thesis statement.
  • The body paragraphs of the rhetorical essay should have a topic sentence. Also, in the paragraph, a thorough analysis should be presented.
  • For writing a satisfactory rhetorical essay conclusion, restate the thesis statement and summarize the main points.
  • Proofread your essay to check for mistakes in the content. Make your edits before submitting the draft.

Following the tips and the essay's correct writing procedure will guarantee success in your academics. 

We have given you plenty of examples of a rhetorical analysis essay. But if you are still struggling to draft a great rhetorical analysis essay, it is suggested to take a professional’s help.

MyPerfectWords.com can assist you with all your academic assignments. The top essay writer service that we provide is reliable. If you are confused about your writing assignments and have difficulty meeting the deadline, get help from custom essay writing online .

Hire our analytical essay writing service today at the most reasonable prices. 

AI Essay Bot

Write Essay Within 60 Seconds!

Nova A.

Nova Allison is a Digital Content Strategist with over eight years of experience. Nova has also worked as a technical and scientific writer. She is majorly involved in developing and reviewing online content plans that engage and resonate with audiences. Nova has a passion for writing that engages and informs her readers.

Get Help

Paper Due? Why Suffer? That’s our Job!

Keep reading

Rhetorical Analysis Essay

Rhetorical Analysis Essay

Rhetorical Analysis Essay Example

Cathy A.

Top 15+ Rhetorical Analysis Essay Examples for Students

Published on: Mar 10, 2023

Last updated on: Jan 29, 2024

rhetorical analysis essay example

People also read

How To Write A Rhetorical Analysis Essay That Stands Out

Rhetorical Analysis Essay Topics & Ideas for Students

Rhetorical Analysis Essay Outline - Tips & Examples

Understanding Ethos, Pathos, Logos - The Three Rhetorical Appeals

Share this article

Writing a rhetorical analysis essay can be tough. You want to engage your reader, but you also need to provide clear and concise analysis of the text. 

It's hard to know where to start, what information is important, and how to make your argument clear. 

Don't fret! We've got you covered. 

In this blog post, we'll give you 15+ Rhetorical analysis essay examples to help you craft a winning essay. Plus, we'll give you some tips on how to make your essay stand out.

So without a further delay, let's start!

On This Page On This Page -->

Good Rhetorical Analysis Essay Examples

Examples help the readers to understand things in a better way. They also help a writer to compose an essay just like professionals.

Here are some amazing rhetorical analysis examples on different topics. Use them as a helping hand to understand the concept and write a good essay.

Rhetorical Analysis Essay Example: AP Language

Rhetorical analysis done in AP Language and Composition is one of the biggest tasks a student can ever get. On the same hand, drafting it in a proper way is also necessary to get good grades.

Look at these rhetorical analysis essay example AP language given below to see how a well-written rhetorical essay is written.

AP Rhetorical analysis essay example

Rhetorical Analysis Essay Example Ap Lang 2020

Rhetorical Analysis Essay Example Ap Lang 2021

Rhetorical Analysis Essay Example AP Lang 2022

Rhetorical Analysis Essay Example AP Lang 2023

These rhetorical analysis essay example college board will help you to win over your panel in no time!

Want to start from the basics? Head over to our Rhetorical essay guide to solidify your base.

Rhetorical Analysis Essay Example: Ted Talk

A rhetorical analysis can be done on nearly anything. Here is a good example of a rhetorical essay in which a ted talk is being analyzed.

Rhetorical Analysis Essay Example: Ethos, Pathos, Logos

The first impression of these three terms sounds just like a conjuration in some kind of a magical story. But in fact, these elements of persuasion were created by Aristotle and have been used for a very long time.

According to Aristotle, they were the primary persuasive strategies that authors should use in their papers. These elements are further elaborated as follows:

  • The ethos appeals to ethics.
  • Pathos appeals to emotions.
  • Logos mean the use of rational thinking.

Here is an example of a rhetorical essay written using these elements.

Understand Ethos,Pathos and Logos to write a compelling essay.

Rhetorical Analysis Essay Example for College

College students often get to write a rhetorical analysis essay. They find it hard to write such an essay because it is a bit more technical than other essay types.

Here is an example of a well-written rhetorical essay for college students.

Comparative Rhetorical Analysis Essay Example

A rhetorical analysis essay can be written to show a comparison between two objects. Here is a compare-and-contrast rhetorical analysis essay example.

Order Essay

Paper Due? Why Suffer? That's our Job!

Visual Rhetorical Analysis Essay Example

The visual rhetorical essay determines how pictures and images communicate messages and persuade the audience. Usually, visual rhetorical essays are written for advertisements. They use strong images to convince the audience to behave in a certain way.

Visual Rhetorical Analysis Essay Example Pdf

Rhetorical Analysis Essay Example: Letter from Birmingham Jail

Here is another good example of a rhetorical essay. Most of us know about the history of “letter from a Birmingham jail”. Read the given example to see how rhetorical analysis is done on it.

Struggling for a similar good topic? Check out our amazing rhetorical essay topics to select the perfect theme for your essay.

Great Influenza: Rhetorical Analysis Essay Example

Influenza has been one of the scariest pandemics the world has faced in history. Here is a rhetorical essay on great influenza.

Great influenza: Rhetorical Analysis Essay Example

Condoleezza Rice Rhetorical Analysis Essay Example

The speech given by Condoleezza Rice has become a classic example of effective oratory. Here is an example of a rhetorical analysis essay on the speech given by Condoleezza Rice at a commencement ceremony.

Rhetorical Analysis Essay Example: Condoleezza Rice’s Commencement Speech

This example explores the effectiveness of Rice's speech and features an in-depth analysis. 

Rhetorical Analysis Essay Example High School

High school essays involve the analysis of different texts and the application of rhetorical tools to those texts. Here is an example that focuses on a high school essay about the effects of television on society. 

Rhetorical Analysis Essay Example (Pdf)

Rhetorical Analysis Essay Example MLA

MLA format is one of the most commonly used formats for essays. Here is an example of a rhetorical essay written in MLA format that focuses on the effectiveness of advertisements. 

MLA Rhetorical Analysis Essay PDF

Rhetorical Analysis Essay Example Outline

Outline helps to organize the ideas and arguments that you want to present in your essay. Here is a sample outline that can help you write an effective rhetorical analysis essay.

Rhetorical Analysis Essay Example Outline sample

Hop on to our rhetorical essay outline guide to learn the step-by-step process of crafting an exemplary outline.

How to Start a Rhetorical Analysis Essay Example

When starting a rhetorical analysis essay, it is important to provide a brief overview of the topic that you are analyzing. This should include the overall message being conveyed, the target audience and the rhetorical devices used in the text. 

Here is a rhetorical analysis introduction example for your ease.

David Suzuki's "The Right Stuff" holds a captivating and enlightening style that we have grown accustomed to from the renowned broadcaster of The Nature Of Things. He commences with an intriguing thought, taken from Is There Life After High School?, that “impressions created in high school are more enduring than those formed at any other time." Undoubtedly, this is something many can relate to.

Thesis Statement Example for Rhetorical Analysis Essay

The thesis statement of a rhetorical analysis essay should explain the primary argument being made in the text. Here is an example of a thesis statement for a rhetorical analysis essay for your ease.

Example of Rhetorical Analysis Thesis Statement

Rhetorical Analysis Essay Example Conclusion

The conclusion of a rhetorical analysis essay is an important part of the overall essay. It should summarize your main points and provide some final thoughts on the topic. 

Here is an example of conclusion for a rhetorical analysis essay for your ease.

Suzuki's essay may overlook the overwhelming question of logistics, which is how can schools—burdened by limited staff and resources—add the formidable subject of sex education to their curriculum? Granted, David Suzuki wrote his essay during a time when educational budgets were in better shape than they are today. Nevertheless, he establishes an salient point that teachers should value their students and present information that captivates their attention.

Download this  Rhetorical Analysis Essay Writing Manual to help gather all the relevant guidance for your rhetorical essay.

Rhetorical Analysis Essay Writing Manual (PDF)

Watch this video to understand how to select Rhetorical analysis essay evidences.

Rhetorical Analysis Essay Writing Tips

To write a rhetorical analysis essay, you must have good writing skills. Writing a rhetorical essay is a technical task to do. This is why many students find it really difficult.

There are the following things that you should do to write a good rhetorical analysis essay. Those important things are as follows:

  • Determine the Rhetorical Strategy

To write a rhetorical essay, the writer needs to follow a specific method for research. The typical research methods used for this particular essay are as follows:

  • Choose a Topic

For any essay type, it is very important to have a good topic. A good topic seeks the readers of attention and convinces them to read the complete essay.

  • Create a Rhetorical Analysis Outline

An outline is an essential part of essay writing. The outline provides a definite structure to the essay and also guides the reader throughout the essay. A  rhetorical analysis outline  has the following elements in it:

  • Introduction
  • Body paragraphs

These three elements let you describe the entire idea of your rhetorical analysis essay. These three elements are further written with the help of sub-elements.

  • Develop a Thesis Statement

The  thesis statement is yet another important part of essay writing. It is the essence of the entire essay. It may be a sentence or two explaining the whole idea of your essay. However, not give background information about the topic.

  • Proofread and Edit

The formal terminology used for essay revision is known as proofreading. To make sure that your essay is error-free, repeat this process more than once.

Now let's wrap up , shall we?

So far we have provided you with the best rhetorical analysis examples that are sure to win over your panel. With our help, you can surely sfe guard your academic success journey in no time!

In case you think you can not write such an essay on your own, consult an essay writing service.

At  CollegeEssay.org , we provide the best rhetorical analysis essay writing service  to every student. We have an extensive team of professional writers who can handle all your essay writing assignments.

We make sure that the content we provide you is 100% unique and help you get the best grade.

So why delay when our essay writer can provide you with a top-notch rhetorical essay right away?

Hire our essay service and avail yourself 50% OFF on your first order today!

Frequently Asked Questions

What are the 3 parts of rhetorical analysis.

The three parts of rhetorical analysis are: 

  • Ethos 
  • Logos 
  • Pathos 

What are the elements of a rhetorical analysis?

The main elements of a rhetorical analysis essay are: 

  • Situation 
  • Audience 
  • Purpose 
  • Medium 
  • Context 

Is there any difference between AP lang rhetorical analysis essay example 2020 and AP lang rhetorical analysis essay example 2021?

Yes, there are differences between 2020 and 2021 AP Language and Composition rhetorical analysis essay examples.

  • In 2020 the essay prompts revolved around various social issues related to public discourse. In 2021 they mainly focused on the ideas of justice or progress. 
  • In 2020 students were encouraged to write a multi-paragraph essay shifting back and forth between creative devices of rhetoric. While in 2021 more emphasis was placed on analyzing how well an author's argument is structured.

Cathy A. (Literature, Marketing)

For more than five years now, Cathy has been one of our most hardworking authors on the platform. With a Masters degree in mass communication, she knows the ins and outs of professional writing. Clients often leave her glowing reviews for being an amazing writer who takes her work very seriously.

Paper Due? Why Suffer? That’s our Job!

Get Help

Keep reading

rhetorical analysis essay example

Legal & Policies

  • Privacy Policy
  • Cookies Policy
  • Terms of Use
  • Refunds & Cancellations
  • Our Writers
  • Success Stories
  • Our Guarantees
  • Affiliate Program
  • Referral Program
  • AI Essay Writer

Disclaimer: All client orders are completed by our team of highly qualified human writers. The essays and papers provided by us are not to be used for submission but rather as learning models only.

rhetorical analysis ap lang example essay

Home — Essay Samples — Literature — Rhetoric — A Rhetorical Analysis of The Best Speech

test_template

A Rhetorical Analysis of The Best Speech

  • Categories: Rhetoric

About this sample

close

Words: 698 |

Published: Jun 13, 2024

Words: 698 | Pages: 2 | 4 min read

Table of contents

Introduction, body paragraph.

Image of Dr. Charlotte Jacobson

Cite this Essay

Let us write you an essay from scratch

  • 450+ experts on 30 subjects ready to help
  • Custom essay delivered in as few as 3 hours

Get high-quality help

author

Verified writer

  • Expert in: Literature

writer

+ 120 experts online

By clicking “Check Writers’ Offers”, you agree to our terms of service and privacy policy . We’ll occasionally send you promo and account related email

No need to pay just yet!

Related Essays

3 pages / 1188 words

1 pages / 513 words

3 pages / 1435 words

2 pages / 871 words

Remember! This is just a sample.

You can get your custom paper by one of our expert writers.

121 writers online

Still can’t find what you need?

Browse our vast selection of original essay samples, each expertly formatted and styled

Related Essays on Rhetoric

Have you ever felt lost in a world that seems to move too fast for you to keep up? In his famous "Lost Ones" speech, renowned author and philosopher, Alan Watts, delves into the existential crisis many individuals face when [...]

Michelle Obama, the former First Lady of the United States, delivered a powerful and inspiring commencement speech at the 2020 “Dear Class of 2020” virtual graduation ceremony. In her speech, Obama addressed the graduating [...]

A big topic in society is education. When searching for a Ted Talk, I found a topic that would be inspiring to educators. The Ted Talk caught my attention was called “Teaching and Being Rachetdemic” by Christopher Emdin. This [...]

Patrick Henry's "Give Me Liberty or Give Me Death" speech, delivered in 1775, is widely regarded as one of the most influential and effective orations in American history. In this speech, Henry rhetorically and passionately [...]

College, for most of us, is a huge step. When you enter college the biggest decision to make is what you will major in. What should you consider when deciding your major? Family, social status, friends, religion, etc. Mark [...]

In William Shakespeare's play "Julius Caesar," Marc Antony delivers a funeral speech that is both captivating and influential. This speech serves as a pivotal moment in the play, as it reveals Antony's cunning use of rhetoric [...]

Related Topics

By clicking “Send”, you agree to our Terms of service and Privacy statement . We will occasionally send you account related emails.

Where do you want us to send this sample?

By clicking “Continue”, you agree to our terms of service and privacy policy.

Be careful. This essay is not unique

This essay was donated by a student and is likely to have been used and submitted before

Download this Sample

Free samples may contain mistakes and not unique parts

Sorry, we could not paraphrase this essay. Our professional writers can rewrite it and get you a unique paper.

Please check your inbox.

We can write you a custom essay that will follow your exact instructions and meet the deadlines. Let's fix your grades together!

Get Your Personalized Essay in 3 Hours or Less!

We use cookies to personalyze your web-site experience. By continuing we’ll assume you board with our cookie policy .

  • Instructions Followed To The Letter
  • Deadlines Met At Every Stage
  • Unique And Plagiarism Free

rhetorical analysis ap lang example essay

IMAGES

  1. ⭐ Ap english language and composition rhetorical analysis essay sample

    rhetorical analysis ap lang example essay

  2. Ap Lang Rhetorical Analysis Essay Examples

    rhetorical analysis ap lang example essay

  3. Guide Rhetorical Analysis Essay with Tips and Examples

    rhetorical analysis ap lang example essay

  4. Methods of Rhetorical Analysis AP English Language and

    rhetorical analysis ap lang example essay

  5. AP Lang Rhetorical Analysis Essay Example: Prompt- Ghandhi Speech

    rhetorical analysis ap lang example essay

  6. Rhetorical Analysis Thesis Statements

    rhetorical analysis ap lang example essay

VIDEO

  1. Rhetorical Analysis Essay Introduction

  2. I Wrote an Ebook!

  3. How Can I Master the Art of Writing a Rhetorical Analysis Essay for AP Lang Q2?

  4. AP Lang Rhetorical Analysis Rubric

  5. How Can I Effectively Annotate for Rhetorical Analysis in AP Lang Q2?

  6. How Can I Effectively Pace My Rhetorical Analysis Essay for AP Lang?

COMMENTS

  1. How to Write the AP Lang Rhetorical Analysis Essay (With Example)

    The AP Lang Rhetorical Analysis Essay is one of three essays included in the written portion of the AP English Exam. The full AP English Exam is 3 hours and 15 minutes long, with the first 60 minutes dedicated to multiple-choice questions. Once you complete the multiple-choice section, you move on to three equally weighted essays that ask you ...

  2. How to Write the AP Lang Rhetorical Essay

    AP Lang Rhetorical Analysis Essay Example Below is a prompt and example for a rhetorical essay, along with its score and what the writer did well and could have improved: The passage below is an excerpt from "On the Want of Money," an essay written by nineteenth-century author William Hazlitt. Read the passage carefully.

  3. AP English Language and Composition Exam Questions

    Download free-response questions from this year's exam and past exams along with scoring guidelines, sample responses from exam takers, and scoring distributions. If you are using assistive technology and need help accessing these PDFs in another format, contact Services for Students with Disabilities at 212-713-8333 or by email at ssd@info ...

  4. PDF Sample Student Responses

    AP English Language and Composition Rhetorical Analysis Free-Response Question (2020) Sample Student Responses . 3 . Sample B [1] On April 9, 1964, United States First Lady Claudia Johnson gave a speech in honor of Eleanor Roosevelt. She spoke to those in attendance of the luncheon, specifically talking to the women within the crowd.

  5. PDF AP English Language and Composition

    1 − Essays earning a score of 1 meet the criteria for the score of 2 but are undeveloped, especially simplistic in their explanation, or weak in their control of language. 0 - Indicates an off-topic response, one that merely repeats the prompt, an entirely crossed-out response, a drawing, or a response in a language other than English.

  6. AP Lang

    for AP Lang: 12 minutes: Read the text and plan out your essay. (TOBI) 6 minutes: Write your introduction paragraph. 18 minutes: Write 2-3 body paragraphs. 2 minutes: Write a quick conclusion. 2 minutes: Proofread and revise your essay. 🎥 Watch: AP Language - Rhetorical Analysis Organization and Timing.

  7. PDF ap06 english lang student samples

    The question directed students to read carefully an excerpt of William Hazlitt's 1827 essay, "On the Want of Money," and to analyze the rhetorical strategies the author uses to develop his position about money. Sample: 2A Score: 8. This essay's control and focus are evident from the first sentence.

  8. AP English Language and Composition: Sample Rhetorical Analysis and

    AP English Language and Composition: Sample Argument Question. The following paragraph is adapted from Mirror for Man, a book written by anthropologist Clyde Kluckhorn in the middle of the twentieth century. Read the passage carefully. Then, write an essay that examines the extent to which the author's characterization of the United States ...

  9. PDF AP English Language and Composition Question 2: Rhetorical Analysis

    AP English Language and Composition Question 2: Rhetorical Analysis (2019) Sample Student Responses 6 Sample HH [1] In the 1930, Indian were oppressed by the British. Britain had a monopoly on the taxation of salt. This unjust tax and control led to the rise of Mohandas "Mahatma" Gandhi as the leader of the Salt March.

  10. PDF HOW TO WRITE: AP Rhetorical Analysis Paragraphs and Essays

    When writing an analysis, it is crucial that you work chronologically through the text. This means that you start at the beginning of the text and work your way through it by discussing what the writer is saying and the effectiveness of the strategies he/she is using at the beginning, middle, and end of the text.

  11. PDF AP English Language and Composition

    AP ® English Language and Composition Sample Student Responses ... Rhetorical Analysis 6 points . On February 27, 2013, while in office, former president Barack Obama delivered the following address dedicating the Rosa Parks st atue in the National ... Write an essay that analyzes the rhetorical choices Obama makes to convey his message. In ...

  12. 3 AP® English Language Rhetorical Essay Strategies

    The AP® English Language rhetorical essay can be nightmare inducing for some AP® students, but there is no need for fear. ... This analysis of the text adds to the textual examples above and continues to bring in new logic from the student. ... which is the main task of this essay. An example of this was in the 2006 AP® English Language ...

  13. PDF How to Write a RHETORICAL ANALYSIS ESSAY Step 1: Full Comprehension of

    AP ELAC Name: _____ How to Write a RHETORICAL ANALYSIS ESSAY Step 1: Full Comprehension of the Text: I. SOAP Analysis of the Prompt - you begin by identifying each part (subject, occasion, etc.,) as much as you can from the PROMPT alone. You may add to your understanding as you read and analyze the passage.

  14. 2020 AP English Exam: How to Write a Rhetorical Analysis Essay

    In light of The College Board recently announcing that this year's AP English Language and Composition exam will be only one question, a 45-minute rhetorical analysis essay, it would probably be a good idea to freshen up on your essay-writing skills.The benefits of doing this won't stop once high-school ends- knowing how to write rhetorical analysis essays will also be extremely helpful ...

  15. PDF Rhetorical Analysis Sample Responses and Scores

    AP English Language and Composition Free-Response Question 2 . Sample Identiier: 2G . Score: 7 This essay presents an adequate argument that moves beyond a score of 6. The student's analysis of Browning's word-choice in her petition to Napoleon ("Browning's first strategy to persuading Napoleon is being respectful and

  16. PDF AP® ENGLISH LANGUAGE AND COMPOSITION

    Sample: 2A Score: 8. This essay effectively identifies and analyzes three of Cesar Chavez's rhetorical choices — striking diction, juxtaposition, and appeals to reader's fundamental moral beliefs — to argue that "nonviolence is the best and most moral way to bring change.". Providing convincing evidence and analysis (for example ...

  17. How to Write a Rhetorical Analysis

    A rhetorical analysis is a type of essay that looks at a text in terms of rhetoric. This means it is less concerned with what the author is saying than with how they say it: their goals, techniques, and appeals to the audience. A rhetorical analysis is structured similarly to other essays: an introduction presenting the thesis, a body analyzing ...

  18. PDF AP English Language and Composition

    AP ® English Language and Composition Sample Student Responses and Scoring Commentary ... Rhetorical Analysis 6 points . ... appeals court judge and then write an essay that analyzed the rhetorical choices Sotomayor made to convey her message about her identity. Students were expected to respond to the prompt with a

  19. Rhetorical analysis essay tips (how I got a 5!)

    The AP rubric lists three ways you can earn the sophistication point: "Explaining the significance or relevance of the writer's rhetorical choices (given the rhetorical situation).". "Explaining a purpose or function of the passage's complexities or tensions.". "Employing a style that is consistently vivid and persuasive.".

  20. 20+ Best Rhetorical Analysis Essay Example & Sample Papers

    Rhetorical Analysis Essay Example AP Lang 2023. Writing a rhetorical analysis for the AP Language and Composition course can be challenging. So drafting it correctly is important to earn good grades. To make your essay effective and winning, follow the tips provided by professionals below: Step #1: Understand the Prompt

  21. Top 15 + Rhetorical Analysis Essay Examples for Students

    Rhetorical Analysis Essay Example: AP Language. Rhetorical analysis done in AP Language and Composition is one of the biggest tasks a student can ever get. On the same hand, drafting it in a proper way is also necessary to get good grades. Look at these rhetorical analysis essay example AP language given below to see how a well-written ...

  22. AP Lang: How do I write a rhetorical analysis essay?

    The conclusion should reflect on your analysis, tie it back to the intro points, and take a more abstract view (i.e. relating your analysis to bigger ideas/concepts). You can do all of this in 3 sentences each, but you won't reach the level of sophistication you need to get high scores.

  23. PDF Sample Student Responses

    AP English Language and Composition Rhetorical Analysis Free-Response Question (2020) Sample Student Responses 1 Sample A [1] Many Americans admired Kennedy and his administration when he was in the White House. He was a loved man. Many share in fond memories of Kennedy up until his death, when the nation grieved for him.

  24. The Great Gatsby: a Rhetorical Analysis

    The Power of Imagery. In The Great Gatsby, Fitzgerald utilizes vivid imagery to create a rich and immersive experience for the reader. For example, he describes Gatsby's parties as "riotous excursions" where "men and girls came and went like moths among the whisperings and the champagne and the stars" (Fitzgerald, 43).

  25. Rhetorical Analysis of "Mother Tongue" by Amy Tan

    Through her use of rhetorical strategies such as anecdotes, appeals to pathos, and contrasting language, Tan effectively conveys the impact of language on identity and the need for a more inclusive understanding of linguistic diversity.

  26. A Rhetorical Analysis of The Best Speech

    This essay aims to conduct a rhetorical analysis of what is widely regarded as the best speech, dissecting the strategies employed by the speaker to achieve maximum impact. By examining the use of ethos, pathos, and logos, alongside the structural elements and language choices, this analysis will elucidate how the speech captivates, convinces ...